Đến nội dung

Hình ảnh

$4(\frac{1}{a}+\frac{1}{b}+\frac{1}{c})+5(a^2+b^2+c^2)\geq 27$


  • Please log in to reply
Chủ đề này có 10 trả lời

#1
nguyenhongsonk612

nguyenhongsonk612

    Thượng úy

  • Thành viên
  • 1451 Bài viết

Cho $a,b,c$ là các số thực dương thỏa mãn $a^3+b^3+c^3=3$. CMR:

$4(\frac{1}{a}+\frac{1}{b}+\frac{1}{c})+5(a^2+b^2+c^2)\geq 27$

P/s: Chúng ta cùng làm nào!!! :namtay 


"...Từ ngay ngày hôm nay tôi sẽ chăm chỉ học hành như Stardi, với đôi tay nắm chặt và hàm răng nghiến lại đầy quyết tâm. Tôi sẽ nỗ lực với toàn bộ trái tim và sức mạnh để hạ gục cơn buồn ngủ vào mỗi tối và thức dậy sớm vào mỗi sáng. Tôi sẽ vắt óc ra mà học và không nhân nhượng với sự lười biếng. Tôi có thể học đến phát bệnh miễn là thoát khỏi cuộc sống nhàm chán khiến mọi người và cả chính tôi mệt mỏi như thế này. Dũng cảm lên! Hãy bắt tay vào công việc với tất cả trái tim và khối óc. Làm việc để lấy lại niềm vui, lấy lại nụ cười trên môi thầy giáo và cái hôn chúc phúc của bố tôi. " (Trích "Những tấm lòng cao cả")

~O) 


#2
nguyenhongsonk612

nguyenhongsonk612

    Thượng úy

  • Thành viên
  • 1451 Bài viết

Cho $a,b,c$ là các số thực dương thỏa mãn $a^3+b^3+c^3=3$. CMR:

$4(\frac{1}{a}+\frac{1}{b}+\frac{1}{c})+5(a^2+b^2+c^2)\geq 27$

Mình nghĩ ra được cách làm rồi!! HEHE!

Theo đề bài ta có:

$a^3+b^3+c^3=3$ => $a^{3}\leq 3\Rightarrow a\leq \sqrt[3]{3}$

Ta C/m BĐT sau là đúng:

$\frac{4}{a^{3}}+5a^{2}\geq 7+2a^3\Leftrightarrow 2a^4-5a^3-7a-4\leq 0\Leftrightarrow a^{3}(2a-5)-(7a+4)\leq 0$

Với $a\leq \sqrt[3]{3}$ => $\left\{\begin{matrix} 2a-5< 0 & & \\ -(7a+4)<0 & & \end{matrix}\right.$ nên BĐT trên đúng.

Tương tự ta có $\frac{4}{b^{3}}+5b^{2}\geq 7+2b^3$ và $\frac{4}{c^{3}}+5c^{2}\geq 7+2c^3$

Nhiệm vụ còn lại là cộng các vế lại và tìm đk dấu "=" xảy ra là xong!!! :)


"...Từ ngay ngày hôm nay tôi sẽ chăm chỉ học hành như Stardi, với đôi tay nắm chặt và hàm răng nghiến lại đầy quyết tâm. Tôi sẽ nỗ lực với toàn bộ trái tim và sức mạnh để hạ gục cơn buồn ngủ vào mỗi tối và thức dậy sớm vào mỗi sáng. Tôi sẽ vắt óc ra mà học và không nhân nhượng với sự lười biếng. Tôi có thể học đến phát bệnh miễn là thoát khỏi cuộc sống nhàm chán khiến mọi người và cả chính tôi mệt mỏi như thế này. Dũng cảm lên! Hãy bắt tay vào công việc với tất cả trái tim và khối óc. Làm việc để lấy lại niềm vui, lấy lại nụ cười trên môi thầy giáo và cái hôn chúc phúc của bố tôi. " (Trích "Những tấm lòng cao cả")

~O) 


#3
Trang Luong

Trang Luong

    Đại úy

  • Thành viên
  • 1834 Bài viết

Mình nghĩ ra được cách làm rồi!! HEHE!

Theo đề bài ta có:

$a^3+b^3+c^3=3$ => $a^{3}\leq 3\Rightarrow a\leq \sqrt[3]{3}$

Ta C/m BĐT sau là đúng:

$\frac{4}{a^{3}}+5a^{2}\geq 7+2a^3\Leftrightarrow 2a^4-5a^3-7a-4\leq 0\Leftrightarrow a^{3}(2a-5)-(7a+4)\leq 0$

Với $a\leq \sqrt[3]{3}$ => $\left\{\begin{matrix} 2a-5< 0 & & \\ -(7a+4)<0 & & \end{matrix}\right.$ nên BĐT trên đúng.

Tương tự ta có $\frac{4}{b^{3}}+5b^{2}\geq 7+2b^3$ và $\frac{4}{c^{3}}+5c^{2}\geq 7+2c^3$

Nhiệm vụ còn lại là cộng các vế lại và tìm đk dấu "=" xảy ra là xong!!! :)

Điều này xảy ra khi giả sử $a\leq b\leq c$

nên k tg tự đc


"Nếu bạn hỏi một người giỏi trượt băng làm sao để thành công, anh ta sẽ nói với bạn: ngã, đứng dậy là thành công"
Issac Newton

#4
nguyenhongsonk612

nguyenhongsonk612

    Thượng úy

  • Thành viên
  • 1451 Bài viết

Điều này xảy ra khi giả sử $a\leq b\leq c$

nên k tg tự đc

Đúng mà bạn ơi! PP uct mà


"...Từ ngay ngày hôm nay tôi sẽ chăm chỉ học hành như Stardi, với đôi tay nắm chặt và hàm răng nghiến lại đầy quyết tâm. Tôi sẽ nỗ lực với toàn bộ trái tim và sức mạnh để hạ gục cơn buồn ngủ vào mỗi tối và thức dậy sớm vào mỗi sáng. Tôi sẽ vắt óc ra mà học và không nhân nhượng với sự lười biếng. Tôi có thể học đến phát bệnh miễn là thoát khỏi cuộc sống nhàm chán khiến mọi người và cả chính tôi mệt mỏi như thế này. Dũng cảm lên! Hãy bắt tay vào công việc với tất cả trái tim và khối óc. Làm việc để lấy lại niềm vui, lấy lại nụ cười trên môi thầy giáo và cái hôn chúc phúc của bố tôi. " (Trích "Những tấm lòng cao cả")

~O) 


#5
songchiviuocmo2014

songchiviuocmo2014

    Binh nhất

  • Thành viên
  • 49 Bài viết

 

 

Cho $a,b,c$ là các số thực dương thỏa mãn $a^3+b^3+c^3=3$. CMR:

$4(\frac{1}{a}+\frac{1}{b}+\frac{1}{c})+5(a^2+b^2+c^2)\geq 27$

P/s: Chúng ta cùng làm nào!!! :namtay 

$a^3+b^3+c^3=3$ \Leftrightarrow $ (a+b+c)(a^2+b^2+c^2-ab-bc-ac)+3abc = 3$
Mà$ ab+bc+ac\leqslant  a^2 +b^2 +c^2$ =>$ 3abc$ \geqslant 3 \Leftrightarrow$ abc$ \geqslant 1
Mà $4(\frac{1}{a}+\frac{1}{b}+\frac{1}{c})+5(a^2+b^2+c^2) = a^2 + b^2 + c^2 + 4(a^2 +\frac{1}{a} +b^2+\frac{1}{b}+c^2+\frac{1}{c})$
Ta lại có $4(a^2 +\frac{1}{a})\geqslant 8(\sqrt{a})$
Tương tự ta có$ P +a^2 + b^2 +c^2$\geqslant$8(\sqrt{ a}+\sqrt{ b}+\sqrt{ c})$
Theo cosi $a^2 + b^2 +c^2 \geq 3\sqrt{a^2b^2c^2}$  \geq$ 24$
dpcm
 



#6
bengoyeutoanhoc

bengoyeutoanhoc

    Trung sĩ

  • Thành viên
  • 124 Bài viết

$a^3+b^3+c^3=3$ \Leftrightarrow $ (a+b+c)(a^2+b^2+c^2-ab-bc-ac)+3abc = 3$
Mà$ ab+bc+ac\leqslant  a^2 +b^2 +c^2$ =>$ 3abc$ \geqslant 3 \Leftrightarrow$ abc$ \geqslant 1
Mà $4(\frac{1}{a}+\frac{1}{b}+\frac{1}{c})+5(a^2+b^2+c^2) = a^2 + b^2 + c^2 + 4(a^2 +\frac{1}{a} +b^2+\frac{1}{b}+c^2+\frac{1}{c})$
Ta lại có $4(a^2 +\frac{1}{a})\geqslant 8(\sqrt{a})$
Tương tự ta có$ P +a^2 + b^2 +c^2$\geqslant$8(\sqrt{ a}+\sqrt{ b}+\sqrt{ c})$
Theo cosi $a^2 + b^2 +c^2 \geq 3\sqrt{a^2b^2c^2}$  \geq$ 24$
dpcm
 

 

Lỗi $\LaTeX$ nhiều quá!

Chả hiểu phần lỗi bạn viết gì! mình giải luôn phần sau vậy nhé! 

Ta có $a^{3}+ b^{3}+ c^{3}-3abc=\left ( a+b+c \right )\left ( a^{2} +b^{2}+c^{2}-ab-ac-bc\right )\Rightarrow 3=3abc+(a+b+c)(a^{2}+b^{2}+c^{2}-ab-ac-bc)$

Mà $a^{2}+b^{2}+c^{2}\geq ab+ac+bc$

Suy ra $3abc\geq 3\Rightarrow abc\geq 1$

Từ đó suy ra $4(\frac{1}{a}+\frac{1}{b}+\frac{1}{c})+5(a^2+b^2+c^2)\geq 4(\frac{9}{a+b+c})+5.\frac{\left ( a+b+c \right )^{2}}{3}=\frac{9}{a+b+c}+\frac{9}{a+b+c}+\frac{9}{a+b+c}+\frac{9}{a+b+c}+\frac{\left ( a+b+c \right )^{2}}{3}+\frac{\left ( a+b+c \right )^{2}}{3}+\frac{\left ( a+b+c \right )^{2}}{3}+\frac{\left ( a+b+c \right )^{2}}{3}+\frac{\left ( a+b+c \right )^{2}}{3}\geq 9\sqrt[9]{\frac{\left ( a+b+c \right )^{8}.9^{4}}{3^{5}}} \geq 9\sqrt[9]{3^{8}.3}=27\rightarrow Q.E.D$

 


Bài viết đã được chỉnh sửa nội dung bởi bengoyeutoanhoc: 22-03-2014 - 19:40


#7
lahantaithe99

lahantaithe99

    Trung úy

  • Thành viên
  • 883 Bài viết

Lỗi $\LaTeX$ nhiều quá!

Chả hiểu phần lỗi bạn viết gì! mình giải luôn phần sau vậy nhé! (Có khi lại là một kiểu khác)

Ta có $a^{3}+ b^{3}+ c^{3}-3abc=\left ( a+b+c \right )\left ( a^{2} +b^{2}+c^{2}-ab-ac-bc\right )\Rightarrow 3=3abc+(a+b+c)(a^{2}+b^{2}+c^{2}-ab-ac-bc)$

Mà $a^{2}+b^{2}+c^{2}\geq ab+ac+bc$

Suy ra $3abc\geq 3$

$\Rightarrow abc\geq 1\Rightarrow \left\{\begin{matrix} \frac{1}{a}+\frac{1}{b}+\frac{1}{c}\geq 3& \\ a^{2}+b^{2}+c^{2}\geq 3& \end{matrix}\right.$

Từ đó suy ra $4(\frac{1}{a}+\frac{1}{b}+\frac{1}{c})+5(a^2+b^2+c^2)\geq 27$ $\rightarrow Q.E.D$

Từ $abc\geqslant 1$ mà suy ra $\frac{1}{a}+\frac{1}{b}+\frac{1}{c}\geqslant 3$ là sai rồi bạn ơi 


Bài viết đã được chỉnh sửa nội dung bởi lahantaithe99: 22-03-2014 - 19:04


#8
bengoyeutoanhoc

bengoyeutoanhoc

    Trung sĩ

  • Thành viên
  • 124 Bài viết

Từ $abc\geqslant 1$ mà suy ra $\frac{1}{a}+\frac{1}{b}+\frac{1}{c}\geqslant 3$ là sai rồi bạn ơi 

Ừ nhỉ! Thanks bạn! Sẽ sửa!



#9
songchiviuocmo2014

songchiviuocmo2014

    Binh nhất

  • Thành viên
  • 49 Bài viết

Mình cũng làm tương tự như bạn trên ấy nhưng mình lợi dụng cái tích nên đưa về dạng đẳng cấp 3 biến mình cô si sẽ ra theo biến tích đó . Thật là tiện



#10
nguyentrungphuc26041999

nguyentrungphuc26041999

    Sĩ quan

  • Thành viên
  • 406 Bài viết

Lỗi $\LaTeX$ nhiều quá!

Chả hiểu phần lỗi bạn viết gì! mình giải luôn phần sau vậy nhé! 

Ta có $a^{3}+ b^{3}+ c^{3}-3abc=\left ( a+b+c \right )\left ( a^{2} +b^{2}+c^{2}-ab-ac-bc\right )\Rightarrow 3=3abc+(a+b+c)(a^{2}+b^{2}+c^{2}-ab-ac-bc)$

Mà $a^{2}+b^{2}+c^{2}\geq ab+ac+bc$

Suy ra $3abc\geq 3\Rightarrow abc\geq 1$

Từ đó suy ra $4(\frac{1}{a}+\frac{1}{b}+\frac{1}{c})+5(a^2+b^2+c^2)\geq 4(\frac{9}{a+b+c})+5.\frac{\left ( a+b+c \right )^{2}}{3}=\frac{9}{a+b+c}+\frac{9}{a+b+c}+\frac{9}{a+b+c}+\frac{9}{a+b+c}+\frac{\left ( a+b+c \right )^{2}}{3}+\frac{\left ( a+b+c \right )^{2}}{3}+\frac{\left ( a+b+c \right )^{2}}{3}+\frac{\left ( a+b+c \right )^{2}}{3}+\frac{\left ( a+b+c \right )^{2}}{3}\geq 9\sqrt[9]{\frac{\left ( a+b+c \right )^{8}.9^{4}}{3^{5}}} \geq 9\sqrt[9]{3^{8}.3}=27\rightarrow Q.E.D$

Chỗ này có vấn đề rồi phải là $abc\leq 1$



#11
bengoyeutoanhoc

bengoyeutoanhoc

    Trung sĩ

  • Thành viên
  • 124 Bài viết

Chỗ này có vấn đề rồi phải là $abc\leq 1$

đúng vậy! Thế thì không dùng được cách này rồi


Bài viết đã được chỉnh sửa nội dung bởi bengoyeutoanhoc: 23-03-2014 - 17:57





1 người đang xem chủ đề

0 thành viên, 1 khách, 0 thành viên ẩn danh